Inequalities in Force of Gravitation between Three Bodies

In summary: It is possible to solve a 4th order polynomial analytically, or...It is possible to solve a 4th order polynomial analytically, or...The sum of the gravitational forces on Planet A is greater than 0.001N.
  • #1
jantdroid
10
0

Homework Statement


Given that the force of gravitation between Planet A (the one in the left side of the drawing), Fa=3000/da2 and the force of gravitation between Planet B and the rocket, Fb= 6000/ db2. Assuming that the three bodies involved is in stationary. What are the distances (ranges) from Planet A where the gravitational force is above 0.001 Newtons?

Homework Equations

The Attempt at a Solution


First, I solved if the Fa>0.001,

Fa=3000/da2
0.001<3000/da2
1732.054< da2

Therefore: 0 < da2 < 1732.054

Then I calculated if Fb>0.001
0.001<6000/db2
2442.75<db2

I subtracted the 2442.75 km from 6 000 000 km because the question asks what distances from planet A where the gravitational force is above 0.001 N.
5997557.25<da

0<da<5997557.25

Not sure if my answer was correct though, I'm still concerned with the middle body which is the rocket.
 

Attachments

  • 10801735_479802048829357_8783249960472937592_n.jpg
    10801735_479802048829357_8783249960472937592_n.jpg
    68.6 KB · Views: 460
Physics news on Phys.org
  • #2
First, I solved if the Fa>0.001, ...
That won't work - no.
Do you need the distance along the line joining planets A and B?

Planets A and B are separated by distance D. At a distance x along the line from A to B, the magnitude of the force of gravity towards A is given by: ##F= F_A-F_B = \cdots## ... you finish it.
Note: you'll need to express ##d_A## and ##d_B## in terms of x.
 
  • Like
Likes jantdroid
  • #3
Simon Bridge said:
That won't work - no.
Do you need the distance along the line joining planets A and B?

Planets A and B are separated by distance D. At a distance x along the line from A to B, the magnitude of the force of gravity towards A is given by: ##F= F_A-F_B = \cdots## ... you finish it.
Note: you'll need to express ##d_A## and ##d_B## in terms of x.

I think so, the question kinda confuses me.

"What are the distances (ranges) from Planet A to B where the gravitational force is above 6 000 000 and I'm concerned about the rocket in the middle too.
 
  • #4
Which question is confusing you, there are several?
The trick with this question is to work out the equation for the sum of the forces - just do that part.
Don't use any of the given values - use letters for them instead.
 
  • #5
Simon Bridge said:
Which question is confusing you, there are several?
The trick with this question is to work out the equation for the sum of the forces - just do that part.
Don't use any of the given values - use letters for them instead.
The one that says find the distance(s) where gravitational force is above 0.001 N
 
  • #6
OK - so did you follow the suggestion?
 
  • #7
Simon Bridge said:
OK - so did you follow the suggestion?
Yah, the equation is on the 4th degree. Not sure how to proceed with it by picking out the critical points and do some test points. :3
 
  • #8
jantdroid said:
Yah, the equation is on the 4th degree. Not sure how to proceed with it by picking out the critical points and do some test points. :3
Are you saying the sum of the forces is a degree 4 polynomial? Because if it is, I think you made a mistake somewhere.
 
  • #9
BiGyElLoWhAt said:
Are you saying the sum of the forces is a degree 4 polynomial? Because if it is, I think you made a mistake somewhere.
Yup, my classmate tried to solved it too still get the 4th degree equation. lol
 
  • #10
Please show your working, with the reasoning behind each step ...
I'm getting a 4th order polynomial that can be reduced to a 2nd order one.
 
Last edited:
  • #11
Simon Bridge said:
Please show your working, with the reasoning behind each step ...
I'm getting a 4th order polynomial that can be reduced to a 2nd order one.
 
  • #12
Simon Bridge said:
Please show your working, with the reasoning behind each step ...
I'm getting a 4th order polynomial that can be reduced to a 2nd order one.

F=FA−FB

Since the force needed is greater than 0.001N

0.001< 3000/da2 - 6000/db2

transposing the 0.001 value to the right side of the equation,
0< 3000/da2 - 6000/db2 - 0.001

Finding the LCD of the solution

0< ( 3000db2-6000da2-0.001da2db2 ) db2 da2

Given that
db= 6 000 000 - da

then,

db2= (6 000 000)2 - 12 000 000 da + da2

substitute all to db and b2

0 < 3000( 3x1013 - 12 000 000 da + da2) - 6000 da2 - 0.001 (3x1013 - 12 000 000 da + da2) da2 ) / da2 (3.6x10 13 - 12 000 000 da + da2 )
 
  • #13
Checking - I made a mistake in my quick once over :)

It's easier to handle big number symbolically:

putting x=d_A, (makes it easier to type) then d_B=D-x: D=6,000,000km
putting G=3000N.km2 means I can write the net force in the direction of planet A as:

$$F=\frac{G}{x^2}-\frac{2G}{(D-x)^2}$$ ... put H=F/G and multiply through by the common denominator:
##\implies H(D-x)^2x^2 = D^2-2Dx-x^2##
##\implies HD^2x^2-2HDx^3+Hx^4= D^2-2Dx-x^2##
##\implies Hx^4-2HDx^3 +(HD^2+1)x^2 + 2Dx - D^2 = 0##

You should double-check my working...
But you get the idea?

The solution we want lies between the planets, which corresponds to the only positive value of x < D.

##F=0.001\text{N} \implies H=(1/3000000) \text{km}^{-2} = 2/D##
... which simplified the equation a bit.

It is possible to solve a 4th order polynomial analytically, or you may prefer a numerical solution via Newton/Raphson.
 
Last edited:
  • Like
Likes jantdroid
  • #14
Simon Bridge said:
Checking - I made a mistake in my quick once over :)

It's easier to handle big number symbolically:

putting x=d_A, (makes it easier to type) then d_B=D-x: D=6,000,000km
putting G=3000N.km2 means I can write the net force in the direction of planet A as:

$$F=\frac{G}{x^2}-\frac{2G}{(D-x)^2}$$ ... put H=F/G and multiply through by the common denominator:
##\implies H(D-x)^2x^2 = D^2-2Dx-x^2##
##\implies HD^2x^2-2HDx^3+Hx^4= D^2-2Dx-x^2##
##\implies Hx^4-2HDx^3 +(HD^2+1)x^2 + 2Dx - D^2 = 0##

You should double-check my working...
But you get the idea?

The solution we want lies between the planets, which corresponds to the only positive value of x < D.

##F=0.001\text{N} \implies H=(1/3000000) \text{km}^{-2} = 2/D##
... which simplified the equation a bit.

It is possible to solve a 4th order polynomial analytically, or you may prefer a numerical solution via Newton/Raphson.
I don't quite get where the "H" came from and what exactly is the answer (distance) from planet a to b
 
  • #15
Simon Bridge said:
x:
Simon Bridge said:
Checking - I made a mistake in my quick once over :)

It's easier to handle big number symbolically:

putting x=d_A, (makes it easier to type) then d_B=D-x: D=6,000,000km
putting G=3000N.km2 means I can write the net force in the direction of planet A as:

$$F=\frac{G}{x^2}-\frac{2G}{(D-x)^2}$$ ... put H=F/G and multiply through by the common denominator:
##\implies H(D-x)^2x^2 = D^2-2Dx-x^2##
##\implies HD^2x^2-2HDx^3+Hx^4= D^2-2Dx-x^2##
##\implies Hx^4-2HDx^3 +(HD^2+1)x^2 + 2Dx - D^2 = 0##

You should double-check my working...
But you get the idea?

The solution we want lies between the planets, which corresponds to the only positive value of x < D.

##F=0.001\text{N} \implies H=(1/3000000) \text{km}^{-2} = 2/D##
... which simplified the equation a bit.

It is possible to solve a 4th order polynomial analytically, or you may prefer a numerical solution via Newton/Raphson.

Here, me trying to apply your method,

Let F> 0.001

0.001 < G (D-x)2 - 2G (x2) / (D-x)2 (x2)

0.001 < G (D2- 2Dx +x2) - 2Gx / (D2-2Dx+x2) (x2)

0.001 < GD2 - 2GDx + Gx2 -2Gx2/ X2D2 - 2Dx3 + x4

0 < GD2- 2GDx - Gx2 / x2D2- 2Dx3 + x4 - 0.001

Finding the LCD:

0 < ( GD2- 2GDx -Gx2 - 0.001 D2x2 - 0.002 Dx3 + 0.001 x4 / x2D2 - 2x3 +x4 ) x2D2 - 2x3 + x4

and now, I am lost. lol
 
  • #16
I don't quite get where the "H" came from...
H is what you get by dividing the first equation through by the common factor of G.
...and what exactly is the answer (distance) from planet a to b
The distance from planet A to planet B is given in the problem statement - I gave this distance the label D, so D=6000000km.

The problem want you to find the distance from planet A to the place where the gravity experienced by the ship is above 0.001N strong.


Let F> 0.001
Put F = 0.001N ... in order for the force to be stronger than this, the ship must be closer to planet A.

There are two places between A and B where the force will have this magnitude, but only one where the direction is towards planet A.

0 < ( GD2- 2GDx -Gx2 - 0.001 D2x2 - 0.002 Dx3 + 0.001 x4 / x2D2 - 2x3 +x4 ) x2D2 - 2x3 + x4
... I didn't bother to check the algebra ... I had 0.001/G = H because it's tidier at the outset.
I chose "H" because it's next in the alphabet from F and G.

Now you have to decide how to solve for x.
You can either use the link I gave you, or find x where F=0 as your starting point for a numerical calculation.
 
  • #17
∠⊆⊆⊕⊕→ℝℝβ
Simon Bridge said:
H is what you get by dividing the first equation through by the common factor of G.
The distance from planet A to planet B is given in the problem statement - I gave this distance the label D, so D=6000000km.

The problem want you to find the distance from planet A to the place where the gravity experienced by the ship is above 0.001N strong.
Put F = 0.001N ... in order for the force to be stronger than this, the ship must be closer to planet A.

There are two places between A and B where the force will have this magnitude, but only one where the direction is towards planet A.

... I didn't bother to check the algebra ... I had 0.001/G = H because it's tidier at the outset.
I chose "H" because it's next in the alphabet from F and G.

Now you have to decide how to solve for x.
You can either use the link I gave you, or find x where F=0 as your starting point for a numerical calculation.
Nice! Thanks for giving me this technique on making equations neat, I'll study how to get that quartic equation right away. I'll get back to you once I've confirmed the answer. Thank you! :D
 

1. What are the three bodies involved in the inequality of force of gravitation?

The three bodies involved in the inequality of force of gravitation are typically referred to as the primary body, the secondary body, and the tertiary body. The primary body is typically the largest and most massive body, while the secondary and tertiary bodies are smaller and have less influence on the inequality of force.

2. How does the mass of each body affect the inequality of force of gravitation?

The mass of each body plays a significant role in the inequality of force of gravitation. The larger the mass of a body, the greater its gravitational pull will be on the other bodies. This means that the primary body will have the strongest influence on the inequality of force, while the secondary and tertiary bodies will have less influence.

3. What other factors besides mass can affect the inequality of force of gravitation?

In addition to mass, the distance between the bodies and their relative positions also play a role in the inequality of force of gravitation. The closer the bodies are to each other, the stronger the gravitational pull will be. Additionally, if the bodies are not aligned in a straight line, their gravitational forces may interact in a more complex manner, resulting in a more unequal distribution of force.

4. What are some real-life examples of inequalities in force of gravitation between three bodies?

A classic example of this phenomenon is the Earth-Moon-Sun system. The Moon orbits around the Earth, while the Earth and Moon both orbit around the Sun. The Sun's massive gravitational force on both the Earth and Moon creates an inequality in their orbits, resulting in tides on Earth and a slightly elliptical orbit for the Moon.

5. How do scientists study and measure the inequality of force of gravitation between three bodies?

Scientists use mathematical equations, such as Newton's Law of Universal Gravitation, to calculate and predict the forces between three bodies. They also use observational data and computer simulations to study the effects of different masses, distances, and positions on the inequality of force of gravitation.

Similar threads

  • Introductory Physics Homework Help
Replies
10
Views
619
Replies
5
Views
2K
  • Introductory Physics Homework Help
Replies
1
Views
701
  • Introductory Physics Homework Help
Replies
10
Views
2K
  • Special and General Relativity
Replies
5
Views
350
  • Introductory Physics Homework Help
Replies
5
Views
5K
  • Introductory Physics Homework Help
Replies
8
Views
1K
  • Introductory Physics Homework Help
Replies
5
Views
2K
  • Introductory Physics Homework Help
Replies
9
Views
786
  • Introductory Physics Homework Help
Replies
14
Views
3K
Back
Top